Изменения

Перейти к: навигация, поиск

Формула включения-исключения

38 байт добавлено, 21:24, 19 октября 2011
Нет описания правки
Тогда исходя из предположения индукции имеем, что
<tex> | B | = \sum \limits_{I \in 2^{N_{-1}}} (-1)^{|I|+1} \left| \bigcap \limits_{ j \in I} A_j \right| </tex>
Тогда из предположения индукции имеем, что <tex> (2) = </tex> <tex> \sum \limits_{I \in 2^{N_{-1}}} (-1)^{|I|+1} \left| \bigcap \limits_{ j \in I} \left( A_j \cap A_n \right) \right| = \sum \limits_{I \in 2^{N_{-1}}} (-1)^{|I|+1} \left| \bigcap \limits_{ j\in I \cup \{ n \} } A_j \right| </tex>
<tex> | A |\!\! = | A_n |+\left( \sum \limits_{I \in 2^{N_{-1}}} (-1)^{|I|+1} \left| \bigcap \limits_{ j \in I} A_j \right| \right) - \left( \sum \limits_{I \in 2^{N_{-1}}} (-1)^{|I|+1} \left| \bigcap \limits_{ j\in I \cup \{ n \} } A_j \right| \right)</tex>
В силу того, что <tex> - \sum \limits_{I \in 2^{N_{-1}}} (-1)^{|I|+1} \left| \bigcap \limits_{ j\in I \cup \{ n \} } A_j \right| = \ \sum \limits_{I \in 2^{N_{-1}}} (-1)^{|I|+2} \left| \bigcap \limits_{ j\in I \cup \{ n \} } A_j \right| \</tex>
Имеем в предыдущей формуле
<tex> | A |\!\!=| A_n |+\left( \sum \limits_{I \in 2^{N_{-1}}} (-1)^{|I|+1} \left| \bigcap \limits_{ j \in I } A_j \right| \right) + \left( \sum \limits_{I \in 2^{N_{-1}}} (-1)^{|I|+2} \left| \bigcap \limits_{ j\in I \cup \{ n \} } A_j \right| \right) </tex> <tex>= \sum \limits_{I \in 2^N} (-1)^{|I|+1} \left| \bigcap \limits_{ j \in I } A_j \right| </tex> .
Равенство справедливо, потому что все наборы <tex> I \in 2^N </tex> можно разбить на три группы :
90
правок

Навигация